Proving Sequence Convergence: Tips & Tricks

Click For Summary
The discussion revolves around proving the convergence of a sequence defined by the relation (n+1)(a_{n+1}-a_n)=n(a_{n-1}-a_n). The original poster suspects the sequence might be alternating or decreasing but struggles to establish this due to a lack of initial terms for induction. A participant suggests defining b_n = a_n - a_{n-1} to analyze the sequence's behavior, resulting in the transformed relation \frac{b_{n+1}}{b_n} = \frac{n}{n+1} < 1. The use of Cantor's Lemma for proving convergence of alternating sequences is mentioned, but the inability to prove the sequence's alternating nature remains a challenge. Overall, the conversation highlights the difficulties in proving convergence without concrete initial values.
estro
Messages
239
Reaction score
0
I know that the sequences meets the following:
(n+1)(a_{n+1}-a_n)=n(a_{n-1}-a_n)
I've got the feeling that this sequence is alternating or decreasing, but I was unable to prove it.
Usually I use induction to prove things about such inductive sequence but in this case I don't have real values t o the first terms of the sequence so I don't have an idea how to even begin inductive prove.
 
Last edited:
Physics news on Phys.org
hi estro! :smile:

(try using the X2 icon just above the Reply box :wink:)
estro said:
(n+1)(a_{n+1}-a_n)=n(a_{n+1}-a_n)

are those two brackets meant to be the same? :confused:

if (n+1)X = nX, then X = 0
 
I have found this question in an old exam, so maybe there is some kind of mistake, I'm not sure.
<br /> (n+1)(a_{n+1}-a_n)=n(a_{n-1}-a_n)<br />
[EDIT] Copied question in a wrong way, now fixed.
 
ah! :rolleyes:

ok … standard way of solving that is to define bn = an+1 - an wink:
 
Still can't see a benefit to define b_m=a_{n+1}-a_n.
I've used to prove convergence of alternating sequences by using Cantor's Lemma in a geometric formulation.
Marked each section [a_{n+1},a_n] and then I proved that each next section smaller and ultimately the length of a sections is going to 0.
But I always had the first terms and used induction to prove sequences is bounded.
But here all these tricks don't work as I can't even prove that this sequence is alternating.
 
If you let b_n = a_n - a_{n-1} then your relation is transformed into \frac{b_{n+1}}{b_n} = \frac{n}{n+1} &lt; 1.
 
Question: A clock's minute hand has length 4 and its hour hand has length 3. What is the distance between the tips at the moment when it is increasing most rapidly?(Putnam Exam Question) Answer: Making assumption that both the hands moves at constant angular velocities, the answer is ## \sqrt{7} .## But don't you think this assumption is somewhat doubtful and wrong?

Similar threads

Replies
4
Views
2K
  • · Replies 1 ·
Replies
1
Views
2K
  • · Replies 3 ·
Replies
3
Views
1K
Replies
6
Views
3K
  • · Replies 4 ·
Replies
4
Views
2K
  • · Replies 9 ·
Replies
9
Views
2K
  • · Replies 2 ·
Replies
2
Views
2K
  • · Replies 34 ·
2
Replies
34
Views
3K
  • · Replies 7 ·
Replies
7
Views
2K
  • · Replies 5 ·
Replies
5
Views
2K